LSAT and Law School Admissions Forum

Get expert LSAT preparation and law school admissions advice from PowerScore Test Preparation.

 Administrator
PowerScore Staff
  • PowerScore Staff
  • Posts: 8916
  • Joined: Feb 02, 2011
|
#74184
Complete Question Explanation

Justify. The correct answer choice is (C)

The structure of the argument is:

..... Premise: Maria won this year’s local sailboat race by beating
..... Sue, the winner in each of the four previous years.

..... Conclusion: We can conclude from this that Maria trained hard.

A quick glance at the argument reveals a gap between the premise and
conclusion—winning does not necessarily guarantee that Maria trained
hard. This is the connection we will need to focus on when considering the
answer choices. To further abstract this relationship, we can portray the
argument as follows:

..... Premise: Maria won (which we could also call “A”)

..... Conclusion: Maria trained hard (which we could also call “B”)

The answer that will justify this relationship is:

..... ..... A :arrow: B

Which is the same as:

..... Maria won :arrow: Maria trained hard

A quick glance at the answer choices reveals that answer choice (C)
matches this relationship (remember, “only if” introduces a necessary
condition). Thus, the structure in this problem matches the first of
the two examples discussed on the previous page. A large number of
Justify questions follow this same model, and you should be prepared to
encounter this form.

Answer choice (A): This answer does not justify the conclusion that Maria
trained hard. The answer does justify the conclusion that Maria trained,
but because this is not the same as the conclusion of the argument, this
answer is incorrect.

Another way of attacking this answer is to use the Justify Formula.
Consider the combination of the following two elements:

..... Premise: Maria won this year’s local sailboat race by beating
..... Sue, the winner in each of the four previous years.

..... Answer choice (A): Sue did not train as hard as Maria trained.

Does the combination of the two elements lead to the conclusion that
Maria trained hard? No, and therefore the answer is wrong.

Answer choice (B): This is a Mistaken Reversal of what is needed (and
therefore the Mistaken Reversal of answer choice (C)). Adding this answer
to the premise does not result in the conclusion. In Justify questions
featuring conditionality, always be ready to identify and avoid Mistaken
Reversals and Mistaken Negations of the relationship needed to justify the
conclusion.

Answer choice (C): This is the correct answer. Adding this answer to the
premise automatically yields the conclusion.

Answer choice (D): Because we do not know anything about Sue except
that she lost, this answer does not help prove the conclusion.
If you are having difficulty understanding why this answer is incorrect,
use the Justify Formula. Consider the combination of the following two
elements:

..... Premise: Maria won this year’s local sailboat race by beating
..... Sue, the winner in each of the four previous years.

..... Answer choice (D): If Sue trained hard, she would win the sailboat
..... race.

The combination of the two creates the contrapositive conclusion that Sue
did not train hard. But, the fact that Sue did not train hard does not tell us
anything about whether Maria trained hard.

Answer choice (E): Because this answer addresses only the relative speed
of the two racers, it fails to help prove that Maria trained hard.

Get the most out of your LSAT Prep Plus subscription.

Analyze and track your performance with our Testing and Analytics Package.